K
Khách

Hãy nhập câu hỏi của bạn vào đây, nếu là tài khoản VIP, bạn sẽ được ưu tiên trả lời.

22 tháng 8 2023

a) Vì \(\pi>1\) nên hàm số \(log_{\pi}x\) đồng biến trên\(\left(0;+\infty\right)\)

Mà \(0,8< 1,2\) nên \(log_{\pi}0,8< log_{\pi}1,2\)

b) Vì \(0,3>1\)  nên hàm số \(log_{0,3}x\)  nghịch biến trên \(\left(0;+\infty\right)\)

Mà \(2<2,1\) nên \(log_{0,3}2>log_{0,3}2,1\)
HQ
Hà Quang Minh
Giáo viên
26 tháng 8 2023

a, Hàm số \(y=log_{\dfrac{1}{2}}x\) có cơ số \(\dfrac{1}{2}< 1\) nên hàm số nghịch biến trên \(\left(0;+\infty\right)\)

Mà \(4,8< 5,2\Rightarrow log_{\dfrac{1}{2}}4,8>log_{\dfrac{1}{2}}5,2\)

b, Ta có: \(log_{\sqrt{5}}2=2log_52=log_54\)

Hàm số \(y=log_5x\) có cơ số 5 > 1 nên hàm số đồng biến trên \(\left(0;+\infty\right)\)

Do \(4>2\sqrt{2}\Rightarrow log_54>log_52\sqrt{2}\Rightarrow log_{\sqrt{5}}2>log_52\sqrt{2}\)

c, Ta có: \(-log_{\dfrac{1}{4}}2=-\dfrac{1}{2}log_{\dfrac{1}{2}}2=log_{\dfrac{1}{2}}\dfrac{1}{\sqrt{2}}\)

Hàm số \(y=log_{\dfrac{1}{2}}x\) có cơ số \(\dfrac{1}{2}< 1\) nên nghịch biến trên \(\left(0;+\infty\right)\)

Do \(\dfrac{1}{\sqrt{2}}>0,4\Rightarrow log_{\dfrac{1}{2}}\dfrac{1}{\sqrt{2}}< log_{\dfrac{1}{2}}0,4\Rightarrow-log_{\dfrac{1}{4}}2< log_{\dfrac{1}{2}}0,4\)

HQ
Hà Quang Minh
Giáo viên
22 tháng 8 2023

\(a,\left(0,3\right)^{x-3}=1\\ \Leftrightarrow x-3=0\\ \Leftrightarrow x=3\\ b,5^{3x-2}=25\\ \Leftrightarrow3x-2=2\\ \Leftrightarrow3x=4\\ \Leftrightarrow x=\dfrac{4}{3}\\ c,9^{x-2}=243^{x+1}\\ \Leftrightarrow3^{2x-4}=3^{5x+5}\\ \Leftrightarrow2x-4=5x+5\\ \Leftrightarrow3x=-9\\ \Leftrightarrow x=-3\)

HQ
Hà Quang Minh
Giáo viên
22 tháng 8 2023

d, Điều kiện: \(x>-1;x\ne0\)

\(log_{\dfrac{1}{x}}\left(x+1\right)=-3\\ \Leftrightarrow x+1=x^3\\ x\simeq1,325\left(tm\right)\)

e, Điều kiện: \(x>\dfrac{5}{3}\)

\(log_5\left(3x-5\right)=log_5\left(2x+1\right)\\ \Leftrightarrow3x-5=2x+1\\ \Leftrightarrow x=6\left(tm\right)\)

f, Điều kiện: \(x>\dfrac{1}{2}\)

\(log_{\dfrac{1}{7}}\left(x+9\right)=log_{\dfrac{1}{7}}\left(2x-1\right)\\ \Leftrightarrow x+9=2x-1\\ \Leftrightarrow x=10\left(tm\right)\)

a: \(log_2\left(mn\right)=log_2\left(2^7\cdot2^3\right)=7+3=10\)

 \(log_2m+log_2n=log_22^7+log_22^3=7+3=10\)

=>\(log_2\left(mn\right)=log_2m+log_2n\)

b: \(log_2\left(\dfrac{m}{n}\right)=log_2\left(\dfrac{2^7}{2^3}\right)=7-3=4\)

\(log_2m-log_2n=log_22^7-log_22^3=7-3=4\)

=>\(log_2\left(\dfrac{m}{n}\right)=log_2m-log_2n\)

D
datcoder
CTVVIP
14 tháng 8 2023

a) \(\log_2\left(mn\right)=\log_2\left(2^7.2^3\right)=\log_22^{7+3}=\log_22^{10}=10.\log_22=10.1=10\)

\(\log_2m+\log_2n=\log_22^7+\log_22^3=7\log_22+3\log_22=7.1+3.1=7+3=10\)

b) \(\log_2\left(\dfrac{m}{n}\right)=\log_2\dfrac{2^7}{2^3}=\log_22^4=4.\log_22=4.1=4\)

\(\log_2m-\log_2n=\log_22^7-\log_22^3=7.\log_22-3\log_22=7.1-3.1=4\)

HQ
Hà Quang Minh
Giáo viên
24 tháng 8 2023

\(a,A=log_23\cdot log_34\cdot log_45\cdot log_56\cdot log_67\cdot log_78\\ =log_28\\ =log_22^3\\ =3\\ b,B=log_22\cdot log_24...log_22^n\\ =log_22\cdot log_22^2...log_22^n\\ =1\cdot2\cdot...\cdot n\\ =n!\)

HQ
Hà Quang Minh
Giáo viên
22 tháng 8 2023

Vì \(\dfrac{1}{e}\simeq0,368< 1\)

\(\Rightarrow y=log_{\dfrac{1}{e}}\left(x\right)\) nghịch biến trên D = \(\left(0;+\infty\right)\)

Chọn C.

0<1/e<1

=>\(log_{\dfrac{1}{e}}\left(x\right)\) nghịch biến 

=>C

18 tháng 8 2023

a) \(log_69+log_64=log_636=2\)

b) \(log_52-log_550=log_5\left(2:50\right)=-2\)

c) \(log_3\sqrt{5}-\dfrac{1}{2}log_550=-1,0479\)

QT
Quoc Tran Anh Le
Giáo viên
22 tháng 9 2023

a)    \({\log _c}b = {\log _a}b.{\log _c}a \Leftrightarrow {a^{{{\log }_c}b}} = {a^{{{\log }_a}b.{{\log }_c}a}} \Leftrightarrow {c^{{{\log }_c}b}} = {\left( {{c^{{{\log }_c}a}}} \right)^{{{\log }_a}b}} \Leftrightarrow b = {a^{{{\log }_a}b}} \Leftrightarrow b = b\) (luôn đúng)

Vậy \({\log _c}b = {\log _a}b.{\log _c}a\)

b)    Từ \({\log _c}b = {\log _a}b.{\log _c}a \Leftrightarrow {\log _a}b = \frac{{{{\log }_c}b}}{{{{\log }_c}a}}\)

HQ
Hà Quang Minh
Giáo viên
23 tháng 8 2023

\(a,a^{log_ab^{\alpha}}=c\Leftrightarrow log_ac=log_ab^{\alpha}\Leftrightarrow c=b^{\alpha}\Rightarrow a^{log_ab^{\alpha}}=b^{\alpha}\\ a^{\alpha log_ab}=c\Leftrightarrow\alpha log_ab=log_ac\Leftrightarrow log_ab^{\alpha}=log_ac\Leftrightarrow b^{\alpha}=c\Rightarrow a^{\alpha log_ab}=b^{\alpha}\\ \Rightarrow a^{log_ab^{\alpha}}=a^{\alpha log_ab}\)

\(b,a^{log_ab^{\alpha}}=a^{\alpha log_ab}\\ \Rightarrow log_ab^{\alpha}=\alpha log_ab\)

HQ
Hà Quang Minh
Giáo viên
22 tháng 9 2023

a) \(y = {\log _a}M \Leftrightarrow M = {a^y}\)

b) Lấy loogarit theo cơ số b cả hai vế của \(M = {a^y}\) ta được

\({\log _b}M = {\log _b}{a^y} \Leftrightarrow {\log _b}M = y{\log _b}a \Leftrightarrow y = \frac{{{{\log }_b}M}}{{{{\log }_b}a}}\)